0
$\begingroup$

Let $f$ be a function defined on the unit square $R = [0,1]^2 \subseteq \mathbf{R}^2$ which is convex and satisfies $\frac{\partial{f}^2 }{\partial{x}\partial{y}} \leq 0$. The last condition is equivalent to the inequality $f(x_1,y_1) + f(x_2,y_2) \geq f(\min\left(x_1,x_2\right), \min\left(y_1,y_2\right)) + f(\max\left(x_1,x_2\right), \max\left(y_1,y_2\right))$ which can be obtained by integrating $\frac{\partial{f}^2 }{\partial{x}\partial{y}}$ over the rectangle.

If we label the vertices of any given rectangle counterclockwise $v_1, \dots, v_4$, starting at the upper right, this is saying that $f(v_2) + f(v_4) \geq f(v_1) + f(v_3)$. Does this property also hold for parallelograms inscribed in $R$ with wlog $v_1 = (1,1), v_3 = (0,0)$?

For $v_2 = (x_2,y_2)$, $v_4 = (x_1,y_1)$, the linear function mapping $R$ to the parallelogram $P$ is

\begin{pmatrix} &x_1 & x_2 \\ &y_1 & y_2 \end{pmatrix}

The differential condition on $P$ can then be written

$(x_1y_2 + y_1x_2)f_{xy} + x_1x_2f_{xx} + y_1y_2f_{yy}$, I don't know that that is necessarily nonpositive.

$\endgroup$

1 Answer 1

2
$\begingroup$

E.g., let $f(x,y):=(x-y)^2+x^2+y^2$ for $(x,y):=R=[0,1]^2$, $v_1:=(1,1)$, $v_2:=(0,t)$, $v_3:=(0,0)$, and $v_4:=(1,1-t)$, where $t\in(0,1/2)$. Then $f$ is convex, $\frac{\partial{f}^2 }{\partial{x}\partial{y}} \le 0$, $v_1v_2v_3v_4$ is a parallelogram inscribed into $R$, but $f(v_2) + f(v_4)\not\ge f(v_1) + f(v_3)$.

$\endgroup$
4
  • $\begingroup$ Thanks, that answers it. I wish to add conditions : $\frac{\partial{f}}{\partial{x}} \geq 0$ and $\frac{\partial{f}}{\partial{y}} \leq 0$. I think the parallelogram inequality holds in this case - proof? $\endgroup$ Jan 17, 2021 at 1:56
  • $\begingroup$ Maybe I'll ask in another post. $\endgroup$ Jan 17, 2021 at 1:58
  • $\begingroup$ @CharlesPehlivanian : If you want these additional conditions to hold as well, add $nx-ny$ to $f(x,y)$ in the example, where $n>0$ is large enough. I guess $n=10$ will suffice. $\endgroup$ Jan 17, 2021 at 2:14
  • $\begingroup$ Thanks, this all works. I'm going to add additional conditions and post as a separate question. $\endgroup$ Jan 17, 2021 at 5:17

Your Answer

By clicking “Post Your Answer”, you agree to our terms of service and acknowledge that you have read and understand our privacy policy and code of conduct.

Not the answer you're looking for? Browse other questions tagged or ask your own question.